¿Cómo funciona la prueba de la conmutatividad del operador con operadores no continuos?

En algunos libros, una prueba de que si dos operadores autoadjuntos A y B comparten una base propia común { ϕ norte } , luego conmutan se da de la siguiente manera:

Para cualquier ϕ norte ,

A B   ϕ norte = a norte   b norte   ϕ norte = B A   ϕ norte ,
y entonces A B B A = 0 .

Entonces, para cualquier vector ψ = norte = 1 C norte   ϕ norte ,

( A B B A )   ψ = ( A B B A ) norte = 1 C norte   ϕ norte = norte = 1   ( A B B A ) C norte ϕ norte = 0

Pero, ¿cómo es válido el tercer paso en la última línea, donde el operador C = ( A B B A ) se mueve a través del signo de suma? C ( límite F norte ) = límite ( C F norte ) solo es valido si C es continuo Pero aquí, mientras C es lineal y autoadjunto, ¡puede que no sea continuo!

Esa es una pregunta que tengo yo mismo. Para los operadores acotados, no debería ser demasiado difícil de probar, pero para los no acotados, no tengo ni idea.
Me pregunto si esto sería más apropiado para math.SE.
Un operador en un espacio de Hilbert (de hecho, más generalmente, un espacio normado) está acotado si y solo si es continuo (de hecho, aún más fuerte: continuo de Lipschitz).

Respuestas (2)

Es posible decir algo más preciso que la respuesta de Martin (sin embargo, eso es correcto). El punto clave es que los operadores autoadjuntos son operadores cerrados . Un operador A : D ( A ) H , con D ( A ) H un subespacio lineal del espacio de Hilbert H se dice que es cerrado si, para toda sucesión de vectores F norte D ( A ) tal que

(1) F norte F H como norte +

y

(2) A F norte gramo H como norte +

entonces F D ( A ) y A F = gramo .

En otras palabras, si ambos D ( A ) { F norte } y { A F norte } convergen, entonces podemos intercambiar el símbolo de límite y A :

A límite norte + F norte = límite norte + A F norte .

Hay varias otras definiciones equivalentes de operadores cerrados, pero esta es la más elemental y se explica por sí misma.

Está claro que los operadores continuos son cerrados pero lo contrario es generalmente falso.

Dentro de la formulación más elemental de QM, los observables están representados por operadores autoadjuntos (generalmente ilimitados y no definidos globalmente) A en un espacio de Hilbert adecuado. Entonces A = A . De la definición de operador adjunto, resulta obvio que A está cerrado. De este modo A está cerrado también. Esta versión más débil de la continuidad permite justificar varias manipulaciones ingenuas como las que mencionaste.

Supongamos ahora que A es autoadjunto (en ese caso D ( A ) es denso) admitiendo un sistema ortonormal completo de vectores propios ϕ norte , dónde A ϕ norte = λ norte ϕ norte (Supongo que el espacio es separable por simple sencillez, pues lo que escribo se mantiene incluso descartando esa hipótesis).

Obviamente ϕ norte D ( A ) y, es posible probar (por ejemplo usando el teorema espectral mencionado por Martin), que ψ = norte C norte ϕ norte D ( A ) si y si norte | C norte λ norte | 2 < + .

Esta última desigualdad, a la vista de la propia definición de base hilbertiana, equivale al hecho de que norte C norte λ norte ϕ norte converge a algún vector ϕ H .

Desde A está cerrado, uno ve fácilmente que:

A ψ = A ( norte C norte ϕ norte ) = norte C norte A ϕ norte = norte λ norte C norte ϕ norte = ϕ .

Hemos probado que:

TEOREMA. Dejar H sea ​​un espacio de Hilbert (complejo separable). Suponer que A es un operador autoadjunto en H , generalmente ilimitado y definido en un subespacio lineal (denso) D ( A ) de H , admitiendo un sistema ortonormal completo de vectores propios ϕ norte , dónde A ϕ norte = λ norte ϕ norte . Si ψ D ( A ) entonces :

A ψ = norte λ norte ϕ norte | ψ ϕ norte .

Ahora, aplicando dos veces el teorema, tenemos un corolario inmediato recordando que, para un par de operadores C , D en H con dominios D ( C ) y D ( D ) respectivamente:

D ( C D ) := { F D ( D ) | D ( F ) D ( C ) } .

COROLARIO. Refiriéndose al teorema anterior, suponga que B : D ( B ) H es otro operador autoadjunto definido en el espacio lineal denso D ( B ) H tal que B ϕ norte = m norte ϕ norte para cada norte . Si ψ D ( A B ) D ( B A ) entonces :

( A B B A ) ψ = 0 .

¡Gracias! Dos preguntas: 1) ¿Qué pasa si A es un operador autoadjunto sin un conjunto completo de vectores propios ortonormales? ¿Se puede cambiar su respuesta anterior para acomodar eso, o eso invalida todo? 2) ¿Qué pasa si A no es autoadjunto?
Con respecto a (1), puede descomponer el espacio de Hilbert como la suma ortogonal directa de la clausura del espacio generado por vectores propios y la ortogonal a ese espacio. Estoy bastante seguro (para estar completamente seguro debo escribir una prueba y no tengo tiempo ahora) que trabajando con vectores contenidos en el subespacio anterior, toda la prueba se mantiene nuevamente. Con respecto a (2) me parece que solo importa el hecho de que A y B son operadores cerrados. Si tienen una base de Hilbert común de vectores propios (donde ahora el λ norte y m norte puede ser complejo) todo funciona.
Nuevamente, también con respecto a (2), debo revisar cada paso para estar completamente seguro...
Pero (con respecto a 2), ¿no se cumple el teorema espectral solo para operadores autoadjuntos? Esto es lo que dice Simon y Reed, capítulo VIII, página 256: "La distinción entre operadores simétricos cerrados y autoadjuntos es muy importante. Es solo para los operadores autoadjuntos que se cumple el teorema espectral y son solo los operadores autoadjuntos los que puede ser exponenciado para dar los grupos unitarios de un parámetro que dan la dinámica en la mecánica cuántica".
@Tim: el teorema espectral se cumple para operadores normales cerrados (consulte el libro de Rudin "análisis funcional"). Con respecto a 2a suponía que A y B son cerrados y que también tienen una base común de vectores propios , en este caso también son operadores normales.
Lo siento, olvidé decir que un operador A Se dice que es normal si A A = A A . Por ejemplo, los operadores autoadjuntos y unitarios son normales.
Incluso operadores normales cerrados A pueden ser exponenciados, ya que el cálculo funcional es válido para ellos. Sin embargo mi i A no es unitario. Estos resultados están bien establecidos desde hace mucho tiempo (ver el libro de Rudin que cité anteriormente) e incluso generalizados a espacios cuaterniónicos de Hilbert, por ejemplo.
¡Entiendo! Reed y Simon están comparando operadores autoadjuntos y operadores cerrados simétricos no autoadjuntos. Dicen que el teorema espectral no se cumple para la segunda clase. Es cierto (incluso si en realidad uno podría usar una descomposición en POVM en lugar de PVM). Con respecto a mi respuesta: si tiene un operador cerrado que admite una base de Hilbert de vectores propios, hay dos posibilidades: los valores propios son reales o no lo son. En el primer caso el operador es autoadjunto. En el segundo no lo es. En ambos casos es normal.
Si tiene dos operadores cerrados con una base común de Hilbert de vectores propios, ellos (son normales y) conmutan, sin importar si los valores propios son reales o complejos.

Como no soy un experto en teoría espectral, esta será solo una respuesta parcial, sin embargo, creo que esta pregunta es matemáticamente mucho más complicada de lo que piensas.

Antes que nada, repasemos el caso de dimensión finita: Tenemos dos matrices hermitianas A , B METRO d y conmutan si y sólo si sus proyecciones espectrales conmutan, es decir, tienen una base propia común. Tanto, tan bien.

Ahora pasamos a un espacio de Hilbert complejo y separable H . Lo primero que hay que tener en cuenta es que para los operadores acotados generales, el término "base de vectores propios" ya no está bien definido. Un operador acotado A B ( H ) podría tener un espectro continuo. Así que primero hagamos algo más simple:

Definición: Un operador C B ( H ) (operador acotado) se dice que es compacto , si se puede aproximar mediante un operador de rango finito (es decir, es el "límite" de los operadores de dimensión finita). Además, un operador compacto se llama autoadjunto si es hermitiano (es decir, ψ , H ϕ = H ψ , ϕ para todos los vectores en el espacio de Hilbert).

Hay caracterizaciones equivalentes más naturales, pero no me importan aquí. Ahora tenemos el teorema espectral:

Teorema: dado un operador compacto autoadjunto A B ( H ) , existe una secuencia de números reales { λ i } i norte acumulando en cero y una secuencia de proyecciones espectrales { PAG i } i norte tal que

A = i = 1 λ i PAG i

Ahora, dado este teorema, siempre podemos usar la prueba anterior. Solo debemos tener en cuenta que cualquier operador lineal acotado es automáticamente continuo (de hecho, un operador lineal está acotado si es continuo).

Ahora vamos a complicarnos más. Para operadores acotados arbitrarios (de nuevo: ¡siempre son continuos!), tenemos un teorema espectral diferente. Para formular esto, necesitamos la noción de una medida con valor de proyección, que nos dará la versión continua del teorema espectral, incorporando aquellas partes del espectro que no son valores propios y por lo tanto no tienen vectores propios (un ejemplo de tal continuo El espectro se puede ver para el Laplaciano: Su espectro es la parte positiva de la línea real y las "funciones propias" serían funciones no integrables al cuadrado - mi i k X ). Entonces definimos:

Definición: Una medida de valor de proyección sobre R es un mapa m : B B ( H ) , dónde B es el sigma-álgebra de Borel sobre los reales, que cumple lo siguiente:

  • m ( R ) = 1
  • para cada ξ , ζ H , ξ , m ( . ) ζ es una medida de valor complejo.

Entonces podemos formular el teorema:

Teorema: dado un operador lineal autoadjunto acotado A B ( H ) , existe una medida con valor de proyección PAG λ en B tal que

A = R λ d PAG λ

Esta es una especie de analogía continua de escribir el operador como una suma de valores propios y proyecciones. En el caso de operadores compactos, nuestro primer teorema simplemente nos dice que la medida d PAG es discreto Ahora, una descomposición de una función espacial de Hilbert en "vectores propios" se vuelve menos obvia: no veo un análogo realmente directo a su prueba anterior. Sin embargo, podemos hacer algo similar. Tenga en cuenta que la cantidad [ A , B ] todavía está bien definido para cada operador A , B B ( H ) , por lo que tiene sentido preguntarse si o no [ A , B ] = 0 . Parece que puedes probar lo siguiente (la prueba se sigue "fácilmente" del teorema espectral):

Proposición: dados dos operadores lineales acotados autoadjuntos A , B , sus medidas espectrales conmutan si los operadores conmutan.

¿Por qué queremos medidas de desplazamiento? Bueno, en este caso, parece que uno debería poder encontrar una medida de valor de proyección conjunta para los dos observables. En este punto, estoy un poco inseguro de cómo formular esto. Supongo que debería definir la medida conjunta como m : B × B B ( H ) es una medida con valor de proyección tal que R d m ( λ ) = PAG A , dónde PAG A es la medida perteneciente a A y de manera similar, si integro sobre la otra variable, obtengo la otra medida. También supongo que la medida en cuestión aquí debería ser solo la medida del producto.

Esto debería ser lo más lejos que podamos llegar con la analogía. Ahora, estabas preguntando específicamente por operadores no continuos. En este caso, debemos considerar operadores ilimitados: estos ocurren naturalmente como hamiltonianos en la mecánica cuántica (sin embargo, prácticamente puede evitarlos considerando siempre la evolución del tiempo). Primero una definición de nuevo:

Definición: Un operador lineal A : D H con D H se dice que es ilimitado , si para cualquier METRO > 0 existe un ψ D tal que A ψ > METRO . Un operador ilimitado se llama hermitiano , si para todos ψ , ϕ D tenemos ψ , A ϕ = A ψ , ϕ . Un operador se llama autoadjunto si, además, el dominio de su operador adjunto también es D .

Eso está bastante claro, pero quería escribir esta definición para aclarar la mayor diferencia entre operadores ilimitados y limitados: mientras que para los operadores limitados, siempre podemos suponer que el dominio es todo el espacio de Hilbert, esto ya no es cierto para operadores ilimitados. De hecho, los operadores ilimitados solo se definen en un subconjunto denso D de todo el espacio de Hilbert. En este punto, la multiplicación de operadores se vuelve problemática: una cantidad como [ A , B ] no está bien definido a priori. Por supuesto, puede convertirlo fácilmente en una expresión bien definida simplemente diciendo que el dominio de [ A , B ] es todos los vectores donde tiene sentido. Sin embargo, en muchos casos, esto significará que el dominio de [ A , B ] es solo { 0 } , lo que hace la expresión [ A , B ] = 0 un poco sin sentido

Resulta que puede usar el mismo teorema espectral que el anterior para operadores ilimitados (la medida del valor de proyección para operadores limitados tendrá soporte compacto, mientras que para operadores ilimitados, esto ya no es cierto). Ahora podemos tener una definición diferente de desplazamiento (ahora está tomada de Reed-Simon VIII.5):

Definición: dos operadores autoadjuntos A , B conmutan si todas sus proyecciones en sus medidas asociadas con valores de proyección conmutan.

Y luego puedes probar el siguiente teorema (nuevamente Reed-Simon):

Teorema: A , B dos operadores autoadjuntos conmutan si sus grupos unitarios de un parámetro conmutan (es decir, mi i t A mi i s B = mi i s B mi i t A para todos s , t ).

Dado esto, podría ser razonable preguntar si existe algo así como una "medida conjunta", pero esto también podría ser irrazonable. Esto es más que mi conocimiento y supongo que tendrías que preguntarle a los matemáticos, si quisieras una exposición más lúcida y completa de estos puntos sutiles...

Como ya se dijo, la última parte está tomada de Michael Reed, Barry Simon: Methods of Modern Mathematical Physics I: Functional Analysis (capítulo VIII).

El resto es de mi cabeza y puede ser defectuoso, los comentarios y sugerencias son muy bienvenidos. Siempre puedo proporcionar más referencias y pruebas, sin embargo, la mayoría de las pruebas (especialmente el teorema espectral) son bastante complicadas.

EDITAR: En general, dado un cálculo del tipo A norte a norte | F norte = norte a norte A | F norte , esto solo se puede hacer para cualquier estado si A está ligado. Encontrará esta situación principalmente en dos contextos: el operador está realmente acotado (entonces, en su mayoría, incluso será compacto y el | F norte son los estados propios) o el operador es ilimitado y el | F norte serán estados propios generalizados. Dado que estos no están en el espacio de Hilbert de todos modos, toda la expresión es solo formal y para ver cómo funciona esto matemáticamente, debe trabajar de manera diferente.

Ahora hay situaciones diferentes a esta. Uno en el que puedo pensar es el hamiltoniano para el oscilador armónico, que no tiene límites, pero solo tiene un espectro de puntos puros. En este caso, para coeficientes generales, H norte a norte | F norte = norte a norte H | F norte es mayormente una tontería en el sentido de que el lado derecho es infinito y el lado izquierdo no está definido, pero al elegir los coeficientes para que decaigan lo suficientemente rápido, el lado derecho tiene sentido. Entonces, puede mostrar fácilmente que es una sucesión de Cauchy, por lo tanto, por completitud converge hacia el lado izquierdo. Sin embargo, esto tiene que mostrarse para cualquier ejemplo particular de una secuencia. En este caso particular, todavía puedes decir que H norte a norte | F norte = norte a norte H | F norte se mantiene simplemente definiendo la relación para que sea verdadera si el lado derecho es infinito y teniendo en cuenta que esto solo significa que el estado no está en el apoyo de H . Sin embargo, no creo que esto sea general de ninguna manera.

¡Gracias por esa respuesta informativa! Mencionó que prácticamente puede evitar los operadores hamiltonianos ilimitados considerando siempre la evolución del tiempo. ¿Puede dar más detalles sobre eso? Además, la prueba de conmutatividad que traje fue solo un ejemplo, pero generalmente cada vez que se usa algo como "A sum f_n = sum A f_n" en QM (y esto es omnipresente), ¿debemos suponer que A es un operador acotado?
a 1): dado un operador esencialmente autoadjunto (no necesariamente acotado) H , siempre existe una evolución temporal unitaria tu ( t ) := mi X pag ( i H t ) . Dado que la evolución del tiempo es unitaria, está en particular acotada para cada t . Entonces, en lugar de trabajar con la ecuación de Schrödinger y los hamiltonianos, también podría trabajar con la evolución temporal y su ecuación diferencial. En realidad, nunca he visto que se haga esto y supongo que es algo inconveniente, ya que esto significa que no se pueden etiquetar los estados propios con energía, etc., pero en principio debería ser factible.
Quiero decir, no puedes evitar los operadores ilimitados para siempre, supongo, pero en muchos contextos, en realidad no necesitas lidiar con los hamiltonianos, si quieres comenzar tu historia con la evolución del tiempo. Para 2), consulte Editar en el texto principal.